Difference between revisions of "2020 AMC 12B Problems/Problem 6"
MRENTHUSIASM (talk | contribs) (→Solution 2: The original solution violates the condition that n>=9. I am adding explanation why violating it is OK.) |
MRENTHUSIASM (talk | contribs) m (→Solution 2) |
||
Line 19: | Line 19: | ||
==Solution 2== | ==Solution 2== | ||
− | Factor out an <math>n!</math> to get | + | Factor out an <math>n!</math> to get <cmath>\frac{(n+2)!-(n+1)!}{n!} = (n+2)(n+1)-(n+1).</cmath> Now, without loss of generality, test values of <math>n</math> until only one answer choice is left valid: |
− | <math>n = 1 \implies (3)(2) - (2) = 4</math>, knocking out <math>\textbf{B} | + | <math>n = 1 \implies (3)(2) - (2) = 4</math>, knocking out <math>\textbf{(B)},\textbf{(C)},</math> and <math>\textbf{(E)}</math>. |
− | + | ||
− | <math>n = 2 \implies (4)(3) - (3) = 9</math>, knocking out <math>\textbf{A}</math> | + | <math>n = 2 \implies (4)(3) - (3) = 9</math>, knocking out <math>\textbf{(A)}.</math> |
This leaves <math>\boxed{\textbf{(D)} \text{ a perfect square}}</math> as the only answer choice left. | This leaves <math>\boxed{\textbf{(D)} \text{ a perfect square}}</math> as the only answer choice left. | ||
− | This solution does not consider the condition <math>n \geq 9.</math> The reason is that, with further testing it becomes clear that for all <math>n</math> | + | This solution does not consider the condition <math>n \geq 9.</math> The reason is that, with further testing it becomes clear that for all <math>n,</math> we get <cmath>(n+2)(n+1)-(n+1) = (n+1)^{2},</cmath> as proved in Solution 1. We have now revealed that the condition <math>n \geq 9</math> is insignificant. |
~DBlack2021 (Solution Writing) | ~DBlack2021 (Solution Writing) |
Revision as of 22:54, 8 June 2021
Problem
For all integers the value of is always which of the following?
Solution 1
We first expand the expression:
We can now divide out a common factor of from each term of this expression:
Factoring out , we get
which proves that the answer is .
Solution 2
Factor out an to get Now, without loss of generality, test values of until only one answer choice is left valid:
, knocking out and .
, knocking out
This leaves as the only answer choice left.
This solution does not consider the condition The reason is that, with further testing it becomes clear that for all we get as proved in Solution 1. We have now revealed that the condition is insignificant.
~DBlack2021 (Solution Writing)
~MRENTHUSIASM (Edits in Logic)
Video Solution
https://youtu.be/ba6w1OhXqOQ?t=2234
~ pi_is_3.14
Video Solution
~IceMatrix
See Also
2020 AMC 12B (Problems • Answer Key • Resources) | |
Preceded by Problem 5 |
Followed by Problem 7 |
1 • 2 • 3 • 4 • 5 • 6 • 7 • 8 • 9 • 10 • 11 • 12 • 13 • 14 • 15 • 16 • 17 • 18 • 19 • 20 • 21 • 22 • 23 • 24 • 25 | |
All AMC 12 Problems and Solutions |
The problems on this page are copyrighted by the Mathematical Association of America's American Mathematics Competitions.